Yahoo Answers is shutting down on May 4th, 2021 (Eastern Time) and the Yahoo Answers website is now in read-only mode. There will be no changes to other Yahoo properties or services, or your Yahoo account. You can find more information about the Yahoo Answers shutdown and how to download your data on this help page.

Question about decomposing an equilateral triangle into two triangles?

What are all possible positive integer pairs (m, n) that validate the picture exactly?

Note:

the smallest solution is (m, n) = (7, 4).

Attachment image

2 Answers

Relevance
  • 6 years ago
    Favorite Answer

    The height of the triangle is h where

    h^2=(2n)^2-n^2=3n^2.

    Also, m^2=h^2+1^2.

    Therefore

    m^2=3n^2+1

    or

    m^2-3n^2=1.

    This is Pell's Equation (see Source). The smallest solution is trivial : (m,n) = (1,0). The smallest non-trivial solution (the fundamental solution) is (2,1). Then comes (7,4) as stated in the question.

    All other solutions can be found from the coupled recurrence relations

    m(k+1)=am(k)+nbn(k)

    n(k+1)=bm(k)+an(k)

    where (a,b)=(m0,n0) is the fundamental solution.

    Here (a,b)=(2,1) so the next solutions are

    m1=2*2+3*1*1=7

    n1=1*2+2*1=4

    m2=2*7+3*1*4=26

    n2=1*7+2*4=15.

  • Duke
    Lv 7
    6 years ago

    Excellent answer and thumbs up for Barry G! All solutions of the Pell's Equation m² - 3n² = 1, starting from (7, 4) exhaust the question.

    Yet I decided to contribute a little bit, because I noticed some points of interest. Imagine not single Cevian as on Rita's picture above, but all 3, whose endpoints divide each side in ratio r = (n - 1)/(n + 1) as shown below. Trivially follows that the green triangle is also equilateral and the areas of the initial and the internal triangles are in ratio

    (r² + r + 1) / (r - 1)²

    http://en.wikipedia.org/wiki/Routh%27s_theorem

    For r = (n - 1)/(n + 1) the above expression yields (3n² + 1)/4, what is integer for all n - odd, but if n is an odd solution of the cited Pell's Equation, then

    3n² + 1 = m² with m - even (m and n have different parity in each solution), so we have in fact proven the following statement:

    Given the conditions in the initial question, the ratio of areas of both equilateral triangles is also integer (and even a perfect square - m²/4) for every solution

    (m, n) of m² - 3n² = 1 with m - even and n - odd.

    First and second such non-trivial solutions are (26, 15) and (362, 209); for (2, 1) both triangles coincide of course.

    Attachment image
Still have questions? Get your answers by asking now.